חשבון אינפיניטסימלי/רציונליים: הבדלים בין גרסאות בדף

תוכן שנמחק תוכן שנוסף
Mathreturn (שיחה | תרומות)
אין תקציר עריכה
Mathreturn (שיחה | תרומות)
אין תקציר עריכה
שורה 36:
{{טענה|
מספר=2|
שם=<math>\forall x\in \mathbb{F} \ \forall n,m \in \mathbb{N} \ \ (x^{m})^n=x^{mn}</math>|
תוכן=יהי <math>m,n \in\mathbb{N}</math>
 
שורה 83:
 
}}
 
{{טענה|
מספר=5|
שם=אי שיוויון ברנולי: <math>\forall n\in N \ \ \forall x\in \mathbb{F} \ \ -1<x \Rightarrow (1+x)^n\ge1+nx</math>|
תוכן=
'''בסיס האינדוקציה:''' <math>\ n=1</math> ואכן מתקיים ש: <math>\ (1+x)^1\ge1+1 \cdot x</math> כלומר: <math>\ 1 + x \ge 1 + x</math> .
 
'''הנחת האינדוקציה:''' נניח את נכונות הטענה עבור <math>\ n=t</math> כלשהו, כלומר נניח ש: <math>\ (1+x)^t \ge 1 + tx</math>
 
נשים לב לכך שמכיוון ש-<math>\ x > -1</math> אז: <math>\ (x+1)>0</math>, ולכן ניתן לכפול את שני אגפי האי-שוויון של ההנחה ולקבל ש: <math>\ (1+x)\cdot(1+x)^t \ge (1+x)\cdot(1+tx)</math>
 
כלומר מתקיים ש- <math>\ (1+x)^{t+1} \ge 1+tx+x+tx^2</math>. בנחה זו נשתמש בהמשך
 
'''צעד האינדוקציה:''' צריך להוכיח את נכונות הטענה עבור <math>\ n=t+1</math> כלומר צריך להוכיח
ש-<math>\ (1+x)^{t+1} \ge 1+(t+1)x</math>:
 
על פי פילוג, <math>\ (1+x)^{t+1} \ge 1+tx+x</math>
 
לפי הנחת האינדוקציה הראינו כי: <math>\ (1+x)^{t+1} \ge 1+tx+x+tx^2>1+tx+x</math> (הביטוי <math>\ tx^2</math> חיובי מאחר ש-<math>\ x^2 \ge 0</math> וגם <math>t \ge 0</math>, ולכן אי השיוויון מתקיים).
}}
 
{{טענה|
מספר=4|
שורה 124 ⟵ 103:
 
כלומר, <math>(x*y^{-1})^n=x^n*(y^{n})^{-1}</math>
}}
 
{{טענה|
מספר=5|
שם=אי שיוויון ברנולי: <math>\forall n\in N \ \ \forall x\in \mathbb{F} \ \ -1<x \Rightarrow (1+x)^n\ge1+nx</math>|
תוכן=
'''בסיס האינדוקציה:''' <math>\ n=1</math> ואכן מתקיים ש: <math>\ (1+x)^1\ge1+1 \cdot x</math> כלומר: <math>\ 1 + x \ge 1 + x</math> .
 
'''הנחת האינדוקציה:''' נניח את נכונות הטענה עבור <math>\ n=t</math> כלשהו, כלומר נניח ש: <math>\ (1+x)^t \ge 1 + tx</math>
 
נשים לב לכך שמכיוון ש-<math>\ x > -1</math> אז: <math>\ (x+1)>0</math>, ולכן ניתן לכפול את שני אגפי האי-שוויון של ההנחה ולקבל ש: <math>\ (1+x)\cdot(1+x)^t \ge (1+x)\cdot(1+tx)</math>
 
כלומר מתקיים ש- <math>\ (1+x)^{t+1} \ge 1+tx+x+tx^2</math>. בנחה זו נשתמש בהמשך
 
'''צעד האינדוקציה:''' צריך להוכיח את נכונות הטענה עבור <math>\ n=t+1</math> כלומר צריך להוכיח
ש-<math>\ (1+x)^{t+1} \ge 1+(t+1)x</math>:
 
על פי פילוג, <math>\ (1+x)^{t+1} \ge 1+tx+x</math>
 
לפי הנחת האינדוקציה הראינו כי: <math>\ (1+x)^{t+1} \ge 1+tx+x+tx^2>1+tx+x</math> (הביטוי <math>\ tx^2</math> חיובי מאחר ש-<math>\ x^2 \ge 0</math> וגם <math>t \ge 0</math>, ולכן אי השיוויון מתקיים).
}}
 
 
{{טענה|